LSAT and Law School Admissions Forum

Get expert LSAT preparation and law school admissions advice from PowerScore Test Preparation.

User avatar
 Dave Killoran
PowerScore Staff
  • PowerScore Staff
  • Posts: 5853
  • Joined: Mar 25, 2011
|
#80602
Complete Question Explanation
(The complete setup for this game can be found here: lsat/viewtopic.php?t=8622)

The correct answer choice is (B)

In the prior question, we discussed the fact that there are still three hospitals remaining to be placed. But, in that question, the hospitals had to be separated in order to “cover” cities 1, 4, and 5, each of which did not contain an institution. This is a Global question, and there is no restriction on the hospitals being together, so it is possible that the three hospitals could all be placed in any of those three cities, and thus we can eliminate cities 1, 4, and 5 from contention. This removes answer choices (A), (C), and (D) from consideration (another way of thinking about those three answers is that according to our diagram, cities 1, 4, and 5 are identical, so none of them is likely to be the correct answer).

Only cities 2 and 6 remain in consideration. City 6 already has a hospital, and there is no restriction on adding two more hospitals (which would leave the remaining hospital to pair with the jail in city 1, 4, or 5), and so city 6 can have three hospitals, and is eliminated from consideration.

City 2 cannot contain all three remaining unplaced hospitals because city 2 cannot contain a jail, and one of the three hospitals must be with the jail that goes in city 1, 4, or 5. Because the maximum number of hospitals in city 2 is thus two, answer choice (B) is the correct answer.
 josuecarolina
  • Posts: 24
  • Joined: Jul 20, 2012
|
#4698
Initially I had a very difficult time understanding the first rule, but got it cleared up from another post. Even after the clarification, this rule is poo. It makes it seem like there can't be a hospital, university or jail in more than one of the cities....But I get it.

Nonetheless, I did not get problem 23.

Thanks in advance!
User avatar
 Dave Killoran
PowerScore Staff
  • PowerScore Staff
  • Posts: 5853
  • Joined: Mar 25, 2011
|
#4704
Hey Josue,

You've run into a tough question here at the end of this section. Even though you answered question #22 correctly, I'm going to include an explanation of that question here because it plays into answering question #23 correctly.

Question #22: Local, Must Be True. The correct answer choice is (D)

From the setup, we know that cities 1, 4, and 5 do not currently contain any of the institutions. The only remaining unplaced institutions are three hospitals and one jail (H, H, H, and J). But, we know that the jail requires a hospital, which effectively creates a JH block, leaving just three entities to cover the three empty cities:


..... ..... ..... ..... ..... J H ..... H ..... H


Thus, although we cannot determine which city the jail is placed in, we can determine that there is a hospital placed in cities 1, 4, and 5. Thus, answer choice (D) must be true and is correct.

Question #23: Global, Must Be True. The correct answer choice is (B)

In the prior question, we discussed the fact that there are still three hospitals remaining to be placed. But, in that question, the hospitals had to be separated in order to “cover” cities 1, 4, and 5, each of which did not contain an institution. This is a Global question, and there is no restriction on the hospitals being together, so it is possible that the three hospitals could all be placed in any of those three cities, and thus we can eliminate cities 1, 4, and 5 from contention. This removes answer choices (A), (C), and (D) from consideration (another way of thinking about those three answers is that according to our diagram, cities 1, 4, and 5 are identical, so none of them is likely to be the correct answer).

Only cities 2 and 6 remain in consideration. City 6 already has a hospital, and there is no restriction on adding two more hospitals (which would leave the remaining hospital to pair with the jail in city 1, 4, or 5), and so city 6 can have three hospitals, and is eliminated from consideration.

City 2 cannot contain all three remaining unplaced hospitals because city 2 cannot contain a jail, and one of the three hospitals must be with the jail that goes in city 1, 4, or 5. Because the maximum number of hospitals in city 2 is thus two, answer choice (B) is the correct answer.

Please let me know if that helps. Thanks!

Get the most out of your LSAT Prep Plus subscription.

Analyze and track your performance with our Testing and Analytics Package.